NGHIỆM và TÍNH CHẤT NGHIỆM của đa THỨC sử DỤNG số PHÚC để GIẢI bài TOÁN về đa THỨC

25 1.7K 3
NGHIỆM và TÍNH CHẤT NGHIỆM của đa THỨC sử DỤNG số PHÚC để GIẢI bài TOÁN về đa THỨC

Đang tải... (xem toàn văn)

Tài liệu hạn chế xem trước, để xem đầy đủ mời bạn chọn Tải xuống

Thông tin tài liệu

CHUYÊN ĐỀ SỬ DỤNG SỐ PHỨC GIẢI CÁC TOÁN VỀ ĐA THỨC Phạm Xuân Thịnh, trường THPT chuyên Hạ Long, Quảng Ninh I MỞ ĐẦU 0.1 Lí chọn đề tài Đa thức đối tượng nghiên cứu nhiều Toán học Ngay từ THCS, THPT đa thức đề cập nhiều chương trình sách giáo khoa sách chuyên khảo Ở bậc học này, đa thức thường nghiên cứu tính chất Đại số, tính chất Số học tính chất Giải tích Đến bậc Đại học sau Đại học, đa thức đối tượng nghiên cứu nhiều ngành Toán học khác Đại số, Lý thuyết số, Hình học, Giải tích, Toán rời rạc, Có lẽ tính quan trọng đa thức Toán học mà toán đa thức thường xuất nhiều kì thi học sinh giỏi, thi học sinh giỏi Quốc gia Quốc tế với mức độ thường đánh giá khó Cái khó toán đa thức nhiều ta không Một vấn đề quan trọng đa thức vấn đề nghiệm Nhiều toán đa thức giải dựa vào việc xử lí nghiệm Tuy nhiên trường số thực, đa thức có nghiệm vô nghiệm Hơn nữa, thật khó để biết đa thức có nghiệm thực hay không Vì việc giải toán đa thức thông qua nghiệm thực trở nên vô khó khăn Nhưng trường số phức lại khác, vấn đề tồn nghiệm đa thức không trở ngại Bởi đa thức khác có nghiệm phức ta xử lí toán thông qua nghiệm phức thuận lợi Vì lí trên, định chọn đề tài:”SỬ DỤNG SỐ PHỨC ĐỂ Các trao đổi với tác giả viết liên hệ qua email : phamxthinh@gmail.com GIẢI CÁC BÀI TOÁN VỀ ĐA THỨC” nhằm nghiên cứu sâu đa thức củng cố thêm kiến thức số phức, phục vụ cho việc dạy học Với đề tài này, hi vọng phần chia sẻ giúp bạn có thêm cách tiếp cận toán đa thức Ngoài ra, thông qua đề tài này, mong muốn nhận ý kiến đánh giá, góp ý trao đổi đồng nghiệp, em học sinh nói riêng tất quan tâm nói chung 0.2 Mục đích nhiệm vụ nghiên cứu Dựa thực tế giảng dạy học tập, mục đích nhiệm vụ nghiên cứu đề tài là: i) Nghiên cứu tính chất sâu đa thức ii) Đề xuất số biện pháp rèn luyện tư duy, lực giải toán kĩ vận dụng số phức để giải toán đa thức như: chia đa thức, toán xác định đa thức, toán đa thức bất khả quy, iii) Tạo tiền đề cho nghiên cứu sử dụng số phức đa thức vào toán tổ hợp 0.3 Đối tượng học sinh Chuyên đề dùng để dạy đối tượng học sinh chuyên toán trường THPT, đặc biệt bồi dưỡng học sinh đội tuyển thi học sinh giỏi Quốc gia Quốc tế Ký hiệu quy ước N : tập số tự nhiên Z : tập số nguyên Q : tập số hữu tỉ R : tập số thực C : tập số phức Q+ (Q− ) : tập số hữu tỉ không âm (không dương) Q∗ : tập số hữu tỉ khác không vp (a) : số mũ cao số nguyên tố p biểu diễn tắc số nguyên a 3 II.NỘI DUNG Kiến thức chuẩn bị Trong mục nêu số kiến thức đa thức (không chứng minh) nhằm phục vụ cho nội dung chuyên đề Giả sử K tập khác rỗng R Kí hiệu K[x] tập đa thức với hệ số thuộc K Định lý Mọi đa thức bậc n (n ≥ 0) với hệ số thực có tối đa n nghiệm thực (tính bội) Định lý Mọi đa thức bậc n (n ≥ 1) có n nghiệm phức (tính bội) Định lý (Định lí Bezout) Số thực a nghiệm đa thức P (x) ∈ R[x] P (x) chia hết cho x − a, hay nói khác đi, tồn đa thức Q(x) ∈ R[x] cho P (x) = (x − a)Q(x), ∀x ∈ R Định nghĩa Đa thức P (x) ∈ K[x] gọi bất khả quy K phân tích thành tích đa thức khác thuộc K[x] Định lý Cho P (x) ∈ Z[x] Khi đó, P (x) bất khả quy Z P (x) bất khả quy Q Định lý Nếu biểu thức đối xứng biến x1 , x2 , , xn nhận giá trị nguyên (hữu tỉ), đa thức đối xứng với hệ số nguyên biến x1 , x2 , , xn nhận giá trị nguyên (tương ứng: hữu tỉ) 2.1 Sử dụng số phức giải toán đa thức Bài toán xác định đa thức, chia hết Ví dụ Tìm tất đa thức P (x) ∈ R[x] thỏa mãn điều kiện P (x).P (2x2 ) = P (2x3 + x), ∀x ∈ R 4 Giải Nếu P (x) ≡ c đa thức hằng, từ giả thiết, ta có c2 = c hay c = c = Xét P (x) đa thức khác Nếu P (x) có nghiệm thực α, 2α3 + α nghiệm P (x) Xảy trường hợp sau • Nếu α = 0, P (0) = Đặt P (x) = xk Q(x) với k ≥ 1, Q(0) = Khi đó, 2k x3k Q(x)Q(2x2 ) = (2x3 + x)k Q(2x3 + x) 2k x2k Q(x)Q(2x2 ) = (2x2 + 1)k Q(2x3 + x) Từ suy Q(0) = Mâu thuẫn ! • Nếu α = 0, ta xét dãy số (un ) xác định u1 = α, un+1 = 2u3n +un , n ≥ Khi đó, P (un ) = 0, ∀n ≥ Hơn nữa, dễ dàng thấy dãy tăng α > giảm α < Điều chứng tở P (x) ≡ Mâu thuẫn ! Vậy P (x) nghiệm thực Do đó, deg(P (x)) = 2n, n ∈ N∗ Vì P (0) = nên tích tất nghiệm P (x) 1, theo tích môđun tất nghiệm Ta chứng minh tất nghiệm P (x) có môđun Thật vậy, điều không đúng, gọi α nghiệm phức có môđun lớn nghiệm P (x), |α| > Khi ấy, từ |2α2 + 1|2 = (2α2 + 1)(2α2 + 1) = 4|α|4 + 2(α2 + α2 ) + = 4|α|2(|α|2 − 1) + 2(α + α)2 + > 1, suy |2α3 +α| = |α|.|2α2 +1| > |α| Mà 2α3 +α nghiệm P (x) Điều mâu thuẫn với cách chọn α Bây từ |α| = |2α3 + α| = 1, ta suy |2α2 + 1| = hay (α + α)2 = 0, tức α = −α Do đó, α số ảo hay α = k.i, k ∈ R Từ |α| = ta có |k| = hay k = ±1 Thành thử, α = ±i Theo đó, P (x) có nghiệm ±i P (x) đa thức với hệ số thực nên bội i −i phải Vì thế, P (x) = (x + i)n (x − i)n = (x2 + 1)n Ví dụ Tìm tất đa thức P (x) ∈ R[x] khác thỏa mãn P (x)P (x + 1) = P (x2 + 1), ∀x ∈ R Giải Trước hết ta thấy P (x) nghiệm thực Thật P (x) có nghiệm thực a, a2 + 1, (a2 + 1)2 + 1, nghiệm Vì a < a2 + < (a2 + 1)2 + < nên P (x) có vô số nghiệm Điều vô lí, P (x) đa thức khác Do đó, deg(P (x)) = 2n, n ∈ N∗ Gọi α nghiệm phức P (x), dễ thấy n α2 + 1, α4 + 1, , α2 + 1, nghiệm P (x) Vì P (x) đa thức khác phải tồn số nguyên dương m, n, m < n cho n m m α2 + = α2 + ⇐⇒ α2 n −2m α2 − = Do α = nên α2 −2 = hay |α| = Như ta chứng minh nghiệm P (x) có môđun Bây gọi α nghiệm P (x), α2 + α2 − 2α + nghiệm Do đó, n m |α| = α2 + = α2 − 2α + = Chú ý α2 + = |α|4 + (α + α)2 − |α|2 + = (α + α)2 nên từ |α2 + 1| = ta suy (α + α)2 = Mặt khác, từ α2 − 2α + 2 = |α|4 − (2 |α|2 + 4)(α + α) + 2(α + α)2 + = − 6(α + α) |α2 − 2α + 2| = ta có α + α = kiện α.α = |α|2 = ta suy √ Kết hợp với √ 3i 3i 1 α = + nghiệm phải có P (x) có nghiệm α = + 2 2 bội Vậy, P (x) = (x − α)n (x − α)n = (x2 − x + 1)n Ví dụ Tìm tất đa thức P (x) ∈ R[x] khác cho P (x)P (x + 1) = P (x2 ), ∀x ∈ R Giải Gọi α nghiệm (thực phức) P (x), dễ dàng thấy n α2 , α4 , , α2 , nghiệm P (x) Do P (x) đa thức nên tồn số nguyên dương m, n(m < n) cho n m m α2 = α2 ⇐⇒ α2 Do đó, α = |α| = n −2m α2 − = 6 i) Nếu α = 0, từ phương trình đề bài, ta cho x = 0, P (1) = 0, suy x = nghiệm ii) Nếu |α| = 1, từ giả thiết, cách cho x = α − 1, ta thấy (α − 1)2 nghiệm, suy |(α − 1)2 | = |(α − 1)2 | = Nếu |(α − 1)2 | = 0, α = 1, |(α − 1)2 | = 1, = (α − 1)2 = |α − 1|2 = (α − 1)(α − 1) = − (α + α), tức α + α = Bây từ giả thiết ta cho x = α2 − 1, nghiệm (α2 − 1)2 , suy |α2 − 1| = |α2 − 1| = • Nếu |α2 − 1| = 0, α2 = hay α = ±i Trường hợp α + α = Mâu thuẫn với α + α = • Nếu |α2 − 1| = 1, = α2 − = α2 − α2 − = − (α + α)2 = Điều vô lí ! Vậy trường hợp, P (x) có nghiệm Do đó, P (x) = axm (x − 1)n , (a = 0; m, n ∈ N∗ ) Thay vào phương trình đề ta a = m = n Vậy P (x) = xn (x − 1)n Ví dụ (VMO, 2015) Cho (fn (x)) dãy đa thức xác định f0 (x) = 2, f1 (x) = 3x, fn (x) = 3xfn−1 (x) + (1 − x − 2x2 )fn−2 (x), ∀n ≥ Tìm tất số nguyên dương n cho fn (x) chia hết cho x3 − x2 + x Giải Vì phương trình x3 − x2 + x = có nghiệm x = x = fn (x) chia hết cho x3 − x2 + x fn (0) = fn + √ i 2 ± √ i nên = Từ giả thiết, ta suy fn (0) = fn−2 (0) fn √ + i 2 = √ 3 + i fn−1 2 √ √ 3 3 + i + − i fn−2 2 2 √ + i 2 Từ f0 (0) = f1 (0) = ta suy fn (0) = n số lẻ Rõ ràng, √ dãy truy hồi tuyến tính cấp có phương trình đặc trưng fn 12 + 23 i √ √ 3 3 3 + i t − + i = t2 − 2 2 √ √ 3 Phương trình có nghiệm t1 = + i t2 = 3.i Từ dễ dàng tính 2 √ √ n √ n 3 i fn + in + i = ( 3) + 2 2 √ √ i π π π π Viết + = cos + i sin i = cos + i sin Khi đó, fn 21 + 23 i = 2 6 2  cos nπ + cos nπ = nπ nπ nπ nπ cos = ⇐⇒ + cos + i sin + sin sin nπ + sin nπ = 6 ⇐⇒ n = 3(2k + 1) Vậy tất số nguyên dương n để fn (x) chia hết cho x3 − x2 + x n = 3(2k + 1), k ∈ N Ví dụ Cho đa thức f (x) ∈ R[x] cho f (x) ≥ 0, ∀x ∈ R Chứng minh tồn đa thức g(x), h(x) ∈ R[x] cho f (x) = g (x) + h2 (x), ∀x ∈ R Giải Gọi hệ số cao f (x) a Vì f (x) ≥ 0, ∀x ∈ R nên a > Hơn nữa, f (x) có nghiệm thực x = a, nghiệm phải có số bội chẵn Do đó, ta giả sử f (x) = a(x − x1 )2k1 (x − x2 )2k2 (x − xm )2km g(x), x1 , x2 , , xm tất nghiệm thực (nếu có) f (x) g(x) đa thức monic nghiệm thực Ngoài ra, f (x) ≥ 0, ∀x ∈ R nên g(x) > 0, ∀x ∈ R Thế g(x) phải có bậc chẵn Đặt deg(g(x)) = 2n, g(x) có 2n nghiệm phức Chú ý g(x) ∈ R[x] nên α nghiệm phức g(x), α nghiệm g(x) với số bội số bội α Gọi α1 , α2 , , αn , α1 , α2 , , αn tất nghiệm phức g(x), g(x) phân tích thành g(x) = [(x − α1 )(x − α1 )]h1 [(x − α2 )(x − α2 )]h2 [(x − αn )(x − αn )]hn Với j = 1, 2, , n ta viết αj = aj + bj i, aj , bj ∈ R (x − αj )(x − αj ) = x2 − (αj + αj )x + αj αj x2 − 2aj x + a2j + b2j = (x − aj )2 + b2j Vì tích tổng bình phương lại tổng bình phương nên g(x) viết thành g(x) = u2 (x) + v (x), ∀x ∈ R, u(x), v(x) ∈ R[x] Bây đặt g(x) = √ m (x − xj ) u(x) h(x) = kj a j=1 √ m a j=1 (x − xj )kj v(x), f (x) = g (x) + h2 (x), ∀x ∈ R Dưới số tập sử dụng số phức để giải Bài (Biên nghiệm) Cho đa thức P (x) = an xn + an−1 xn−1 + · · · + ax + a0 ∈ Z[x], an = Đặt M = max {|ai | : i = 0, 1, 2, , n − 1} Chứng minh α M nghiệm P (x), |α| < + |an | Bài Tìm tất số nguyên dương n cho đa thức f (x) = (x + 1)n + xn + chia hết cho g(x) = x2 + x + Bài Tìm tất đa thức P (x) ∈ R[x] cho P (x)P (x − 3) = P (x2 ), ∀x ∈ R Bài Tìm tất đa thức P (x) ∈ R[x] cho P (x)P (x + 1) = P (x2 + x + 1), ∀x ∈ R Bài Cho đa thức P (x), Q(x), R(x), S(x) ∈ R[x] thỏa mãn P (x5 ) + x.Q(x5 ) + x2 R(x5 ) = (1 + x + x2 + x3 + x4 )S(x), ∀x ∈ R Chứng minh P(x) chia hết cho x − 9 2.2 Chứng minh đa thức bất khả quy Để chứng minh đa thức P (x) ∈ Z[x] bất khả quy Z ta sử dụng phương pháp phản chứng, tức giả sử P (x) khả quy Khi ấy, tồn đa thức f (x), g(x) ∈ Z[x] có bậc lớn cho P (x) = f (x).g(x), ∀x ∈ R Từ cách sử dụng kết đa thức, đặc biệt kết nghiệm đa thức, ta tìm điều vô lí hay mâu thuẫn kết thúc chứng minh Sau ví dụ minh họa Trong suốt mục không nói thêm, ta hiểu bất khả quy (khả quy) nghĩa bất khả quy (khả quy) Z Ví dụ (JMO,1999) Cho n số nguyên dương Chứng minh đa thức P (x) = (x2 + 12 )(x2 + 22 ) (x2 + n2 ) + bất khả quy Giải Giả sử f (x) khả quy Z, tức f (x) = g(x).h(x), với g(x), h(x) ∈ Z[x] có bậc lớn Thế g(k.i).h(k.i) = f (k.i) = 1, ∀k ∈ Z, ≤ |k| ≤ n Vì g(x), h(x) ∈ Z[x] nên g(k.i) h(k.i) có dạng a + b.i với a, b ∈ Z, suy |g(k.i)|2 |h(k.i)|2 số tự nhiên Do đó, từ |g(k.i)|2 |h(k.i)|2 = suy |g(k.i)| = h(k.i) = = g(k.i) = g(k.i) = g(−k.i) g(k.i) Điều dẫn tới đa thức h(x) − g(−x) có 2n nghiệm Hơn nữa, đa thức có bậc nhỏ 2n nên h(x) = g(−x), ∀x Như thế, f (x) = g(x).g(−x) đó, f (0) = [g(0)]2 , tức (n!)2 + = [g(0)]2 Dễ thấy điều Vậy điều giả sử sai hay f (x) đa thức bất khả quy Ví dụ (IMO,1993) Chứng minh đa thức f (x) = xn + 5xn−1 + bất khả quy Z Giải Giả sử f (x) đa thức khả quy, tức tồn g(x), h(x) ∈ Z[x] có bậc lớn cho f (x) = g(x).h(x) Vì g(−5).h(−5) = f (−5) = nên |g(−5)| = |h(−5)| = Giả sử |g(−5)| = Đặt k = deg(g(x)) gọi α1 , α2 , , αk nghiệm phức g(x), g(x) = (x − α1 )(x − α2 ) (x − αk ) 10 Từ suy |(α1 + 5)(α2 + 5) (αk + 5)| = |g(−5)| = Vì f (αi ) = nên αin−1 (αi + 5) = −3, ∀i = 1, k Do đó, |α1 α2 αk |n−1 |(α1 + 5)(α2 + 5) (αk + 5)| = 3k Để ý |α1 α2 αk | = |g(0)| g(0).h(0) = f (0) = nên |g(0)| = |g(0)| = Vì 3k > nên |g(0)| > 1, suy |g(0)| = Khi ấy, 3n−1 = 3k hay k = n − =⇒ deg(h(x)) = 1, tức f (x) phải có nghiệm nguyên x = a Điều xảy Vậy f (x) đa thức bất khả quy Ví dụ Cho a, m, n số nguyên dương số nguyên tố p < a − Chứng minh đa thức P (x) = xm (x − a)n + p bất khả quy Giải Giả sử P (x) khả quy Thế tồn đa thức f (x), g(x) ∈ Z[x] có bậc lớn cho P (x) = f (x)g(x), ∀x ∈ R Vì p = P (0) = f (0)g(0) nên giả sử f (0) = Đặt k = deg(f (x)) gọi x1 , x2 , , xk nghiệm phức f (x) Do P (x) monic nên giả sử f (x) monic Khi ấy, f (x) = (x − x1 )(x − x2 ) (x − xk ) Từ suy |x1 x2 xk | = |f (0)| = Vì xi , (1 ≤ i ≤ k) nghiệm n P (x) nên xm i (xi − a) = −p Do đó, n m n m n k |f (a)|n = |(a−x1 )(a−x2 ) (a−xk )|n = |xm (x1 −a) |.|x2 (x2 −a) | |xk (xk −a) | = p Từ suy |f (a)| = p k = n Theo đó, f (a) − f (0) ∈ {p − 1, p + 1, −p + 1, −p − 1} Mà a | f (a) − f (0) nên a | p − a | p + Nhưng hai tình không xảy ra, < p − < p + < a Vậy điều giả sử sai hay P (x) phải đa thức bất khả quy Ví dụ Cho p số nguyên tố an an−1 a1 a0 , an > biểu diễn p hệ thập phân Chứng minh đa thức P (x) = an xn + an−1 xn−1 + · · · + a1 x + a0 bất khả quy Giải Giả sử ngược lại P (x) đa thức khả quy Khi tồn Q(x), H(x) ∈ Z[x] có bậc lớn cho P (x) = Q(x).H(x) Vì P (10) = an 10n + an−1 10n−1 + · · · + a1 10 + a0 = p 11 nên |Q(10)| = |H(10)| = Giả sử |Q(10)| = gọi x1 , x2 , , xk , với k = deg(Q(x)) tất nghiệm phức Q(x), Q(x) = a(x − x1 )(x − x2 ) (x − xk ), a ∈ Z, a = Đặt M = max {a0 , a1 , , an−1 } , dễ dàng chứng M minh |xi | ≤ + ≤ + < Từ suy an = |Q(10)| = |a|.|10 − x1 |.|10 − x2 | |10 − xk | > Điều vô lí chứng tỏ P (x) bất khả quy Ví dụ Cho n ∈ N∗ Chứng minh đa thức f (x) = (x2 − 7x + 6)2n + 13 bất khả quy Giải Giả sử f (x) đa thức khả quy, tức tồn g(x), h(x) ∈ Z[x] có bậc lớn cho f (x) = g(x).h(x) Gọi xi , i = 1, k tất nghiệm phức g(x), k = deg(g(x)) Khi ấy, g(x) = (x − x1 )(x − x2 ) (x − xk ) Từ suy Do đó, |g(1)| = |(x1 − 1)(x2 − 1) (xk − 1)| , |g(6)| = |(x1 − 6)(x2 − 6) (xk − 6)| (x21 − 7x1 + 6)(x22 − 7x2 − 6) (x2k − 7xk + 6) 2n = g 2n (1).g 2n (6) Vì xi , i = 1, k nghiệm g(x) nên (x2i − 7xi + 6)2n = −13 Thành thử, (x21 − 7x1 + 6)(x22 − 7x2 − 6) (x2k − 7xk + 6) 2n = 13k Như thế, g 2n (1).g 2n (6) = 13k Vì 13 số nguyên tố nên k = 2n, suy |g(1)| |g(6)| = 13 Nhưng | g(6) − g(1) nên g(6) = g(1) Suy 13 = g (1) Vô lí ! Chú ý Ta có vài ý sau toán i) Bài toán kết mạnh toán kì thi VMO năm 2014: ”Cho đa thức P (x) = (x2 − 7x + 6)2n + 13, n ∈ N∗ Chứng minh P (x) phân tích thành tích n+ đa thức khác với hệ số nguyên.” 12 ii) Nếu thay 13 số nguyên tố p cho p ≡ ±1 (mod 5), kết không thay đổi Ví dụ Cho n số nguyên dương p số nguyên tố Chứng minh đa thức f (x) = xp − x + pn bất khả quy Giải Giả sử f (x) khả quy, tức tồn đa thức g(x), h(x) ∈ Z[x] có bậc lớn cho f (x) = g(x).h(x), ∀x ∈ R Từ pn = f (0) = g(0).h(0) suy |g(0)| = pa |h(0)| = pb Không tính tổng quát, ta coi ≤ a ≤ b Viết g(x) = xk + ak−1 xk−1 + · · · + a1 x + a0 h(x) = xm + bm−1 xm−1 + · · · + b1 x + b0 , a0 = ±pa b0 = ±pb Bằng cách đồng hệ số, ta có −1 = a1 b0 + a0 b1 = pa (±b1 ± a1 pb−a ) Suy a = đó, |g(0)| = Gọi x1 , x2 , , xk nghiệm g(x), g(x) = (x − x1 )(x − x2 ) (x − xk ) =⇒ |x1 x2 xk | = |g(0)| = Từ ta thấy phải tồn i (1 ≤ i ≤ k) cho |xi | ≤ Do xi nghiệm f (x) nên pn = |xi − xpi | ≤ |xi | + |xi |p ≤ Điều xảy p = n = Khi đó, f (x) = x2 − x + Nhưng đa thức khả quy Vậy điều giả sử sai hay f (x) phải đa thức bất khả quy Ví dụ Cho p số nguyên tố a số nguyên không chia hết cho p Chứng minh P (x) = xp − x + a bất khả quy Giải Giả sử P (x) khả quy Khi tồn Q(x), H(x) ∈ Z[x] có bậc lớn cho P (x) = Q(x).H(x) Đặt k = deg(Q(x)) gọi x1 , x2 , , xk tất nghiệm phức Q(x) Ta có xpi = xi − a, ∀i = 1, k Do xp1 + xp2 + · · · + xpk = x1 + x2 + · · · + xk − k.a Vì Q(x) ∈ Z[x] nên theo định lí Vieta, đa thức đối xứng x1 , x2 , , xk số nguyên Do tất đa thức đối xứng biến 13 x1 , x2 , , xk số nguyên Nói riêng, x1 +x2 +· · ·+xk ∈ Z xp1 +xp2 +· · ·+xpk ∈ Z Theo định lí nhỏ Fermat, ta có (x1 + x2 + · · · + xk )p ≡ x1 + x2 + · · · + xk (mod p) Mặt khác, ta lại có xp1 + xp2 + · · · + xpk = (x1 + x2 + · · · + xk )p − 0≤i1 ,i2 , ,ik |a1 | + |a2 | + · · · + |an | Chứng minh P (x) bất khả quy Z Giải Giả sử P (x) khả quy Khi tồn Q(x), H(x) ∈ Z[x] có bậc lớn hay cho P (x) = Q(x).H(x) Vì |P (0)| = |a0 | số nguyên tố nên |Q(0)| = |H(0)| = Giả sử |Q(0)| = Gọi x1 , x2 , , xk tất nghiệm phức Q(x), k = deg(Q(x)), Q(x) = a.(x − x1 )(x − x2 ) (x − xk ), với a | an Vì ≤ Do đó, tồn i, ≤ i ≤ k cho |xi | ≤ |Q(0)| = nên |x1 |.|x2 | |xk | = |a| Rõ ràng, xi nghiệm P (x) nên P (xi ) = hay a0 = −xi a1 + a2 xi + · · · + an xin−1 Từ suy |a0 | = |xi | a1 + a2 xi + · · · + an xin−1 ≤ |a1 | + |a2 | + · · · + |an | Điều mâu thuẫn với giả thiết Ví dụ Cho p số nguyên tố, k, h ∈ N∗ cho f (x) = k.xp − h đa thức bất khả quy p h ∈ / Q Chứng minh k Giải Ta cần xét (h, k) = Giả sử f (x) = g(x).h(x), với g(x), h(x) ∈ Z[x] có bậc lớn Đặt m = deg(g(x)), ≤ m < pvà gọi ε1 , ε2 , , εm 14 nghiệm phức g(x) Khi đó, εi , ∀i = 1, m bậc p g(x) = a(x − ε1 )(x − ε2 ) (x − εm ), với a ước dương k Ta có h k h = |f (0)| = |g(0)|.|h(0)| = ab|ε1 |.|ε2| |εm |, b = |h(0)| ước dương h Từ suy p p p h = a b |εp1 |.|εp2| |εpm | p p = a b h k m k = c.au aum bv(p−m) cm dp−m = ap bp Vì h = d.bv a, b, c, d đôi nguyên tố nên   p m=1 a = aum       b = bp = bv(p−m) h=1 =⇒ =⇒   c=1 k = ap cm =       p−m d =1 d=1 Suy hp−m k m = ap bp Đặt Thành thử, p h = ∈ Q Mâu thuẫn ! k a Ví dụ 10 Cho p số nguyên tố dạng 4k + Chứng minh đa thức f (x) = (x2 + 1)n + p bất khả quy Giải Giả sử ngược lại f (x) = g(x)h(x), g(x), h(x) ∈ Z[x] có bậc lớn Ta có p = h(i) = g(i)h(i) =⇒ p2 = |g(i)|2.|h(i)|2 (1) Vì số |g(i)|2 |h(i)|2 có dạng a2 + b2 với a, b ∈ Z p số nguyên tố dạng 4k + nên xảy trường hợp |g(i)|2 = |h(i)|2 = p Do đó, ta giả sử |g(i)| = Đặt k = deg(g(x)) gọi x1 , x2 , , xk nghiệm g(x), từ f (x) monic ta suy g(x) Hơn nữa, ta có g(x) = (x − x1 )(x − x2 ) (x − xk ) Khi đó, = |g(i)|2 = g(i)g(i) = g(i)g(−i) = (x21 + 1)(x22 + 1) (x2k + 1) 15 Để ý xj , j = 1, 2, , k nghiệm f (x) nên x2j + có = (x21 + 1)n (x22 + 1)n (x2k + 1)n = (−p)k n = −p Do đó, ta Đây điều vô lí ! Ví dụ 11 (Việt Nam TST, 2013) Tìm tất số nguyên n > số nguyên tố p cho đa thức f (x) = xn − px + p2 khả quy Z Giải Giả sử f (x) = g(x)h(x), g(x), h(x) ∈ Z[x] có bậc k, m ≥ (k + m = n) Từ p2 = f (0) = g(0).h(0) suy |g(0)| ∈ {1; p; p2 } Nhưng thực tế ta cần quan tâm đến trường hợp |g(0)| = |g(0)| = p Gọi x1 , x2 , , xk nghiệm phức g(x), g(x) = (x − x1 )(x − x2 ) (x − xk ) • Nếu |g(0)| = 1, |x1 x2 xk | = Khi đó, xni = p(xi − p), ∀i = 1, k nên = |x1 x2 xk |n = pk |(x1 − p)(x2 − p) (xk − p)| = pk |g(p)| Đây điều vô lí ! • Nếu g(0) = h(0) = ±p, từ |x1 x2 xk | = |g(0)| = p ta suy pn = |x1 x2 xk |n = pk |(x1 − p)(x2 − p) (xk − p)| = pk |g(p)| Thành thử, |g(p)| = pm Khi đó, ta có |h(p)| = pk Do |g(0)| = |h(0)| = p nên g(−p) h(−p) chia hết cho p Mà g(−p).h(−p) = f (−p) = p2 + (−p)n−2 nên g(−p) = a.p h(−p) = b.p, a, b ∈ Z cho ab = + (−p)n−2 Ta có g(p) + g(−p) = 2g(0) + A.p2 g(p) + g(−p) = a.p + pm h(p) + h(−p) = 2g(0) + B.p2 h(p) + h(−p) = b.p + pk với A, B ∈ Z Từ suy m ≥ k ≥ 2,  a.p − 2g(0) p2 a ≡ −2 (mod p) a ≡ (mod p) =⇒ b.p − 2g(0) p2 b ≡ −2 (mod p) b ≡ (mod p) 16 Cả hai trường hợp ta có ab ≡ (mod p), suy + (−p)n−2 ≡ (mod p) hay p = D đó, a b số chẵn nên ab hay + (−2)n−2 Nhưng điều không xảy n = m + k ≥ Như bắt buộc phải có k = m = 1, tức f (x) phải có nghiệm nguyên x0 Từ xn0 = p(x0 − p) suy x0 p hay x0 = t.p Do đó, tn pn−2 = t − Phương trình có nghiệm t = −1, p = 2, n = Thử lại ta thấy n = 3, p = thỏa mãn đề tất giá trị cần tìm Ví dụ 12 Cho p số nguyên tố có dạng 4k + a, b số nguyên cho {vp (a), vp (b − 1)} = Chứng minh đa thức f (x) = x2p + a.x + b bất khả quy Giải Giả sử f (x) khả quy, tức f (x) = g(x).h(x) với g(x), h(x) ∈ Z[x] có bậc lớn Ta có |f (i)|2 = |b − + a.i|2 = a2 + (b − 1)2 Đặt b − = c.pu a = d.pv (với c, d ∈ Z; (c, p) = (d, p) = u, v ∈ N∗ ; {u, v} = 1) Khi a2 + (b − 1)2 = p2 (c2 + d2 p2v−2 ) a2 + (b − 1)2 = p2 (c2 p2u−2 + d2 ) Vì (c, p) = (d, p) = p số nguyên tố có dạng 4k+3 nên c2 +d2 p2v−2 c2 p2u−2 + d2 không chia hết cho p Do đó, vp |f (i)|2 = Rõ ràng |g(i)|2 , |h(i)|2 ∈ N∗ Do đó, từ |g(i)|2 |h(i)|2 = |f (i)|2 p số nguyên tố có dạng 4k + nên xảy vp |g(i)|2 = vp |h(i)|2 = Giả sử vp |g(i)|2 = vp |h(i)|2 = Đặt k = deg(h(x)) gọi x1 , x2 , , xk tất nghiệm phức h(x) dễ thấy h(x) monic h(x) = kj=1 (x − xj ) Ta có h(i) = kj=1 (i − xj ), suy p k k j=1 (x2j + 1) = |h(i)| =⇒ (x2j + 1) j=1 = |h(i)|2p p−1 l 2l a b−1 Từ suy Cp x − x − p l=1 p p = p.g(xj ), ∀j = 1, k Như thế, ta có Để ý f (x) = (x2 + 1) − p.g(x) với g(x) = p x2j + p p k (x2j j=1 + 1) k = |h(i)| 2p =p k g(xj ) , j=1 (∗) 17 Vì h(x) ∈ Z[x] nên theo định lí Vieta đa thức đối xứng x1 , x2 , , xk số nguyên Do đó, tất đa thức đối xứng với hệ số nguyên x1 , x2 , , xk k số nguyên Dễ thấy g(x) ∈ Z[x] nên j=1 g(xj ) số nguyên Thành thử từ (∗) suy |h(i)|2p chia hết cho p Điều vô lí ! Ví dụ 13 Chứng minh đa thức f (x) = (x2 + x)2 + bất khả quy với n ∈ N∗ n Giải Giả sử f (x) = g(x).h(x), g(x), h(x) ∈ Z[x] có bậc lớn √ giả thiết g(x) √ h(x) Đặt ε = √ Vì f (x) monic nên a + 3b.i c + 3d.i −1 + 3i , g(ε) = h(ε) = , a, b, c, d ∈ Z 2k 2k (a, b, 2) = (c, d, 2) = Từ = f (ε) = g(ε).h(ε) =⇒ a2 + 3b2 c2 + 3d2 = 4k+m+1 Do phương trình x2 + 3y = nghiệm nguyên nên a2 + 3b2 = 4u c2 + 3d2 = 4v (u + v = k + m + 1) Ta giả sử u ≤ v Khi xảy trường hợp sau • Nếu u = 0, a2 = 1, b = Khi v > c, d phải có lẻ, suy c2 + 3d2 ≡ (mod 8) =⇒ v = Từ ta có k + m = ⇐⇒ k = m = hay |g(ε)| = |h(ε)| = • Nếu u > 0, v > Lập luận tương tự trên, ta u = v = hay k + m = Do k = 1, m = k = 0, m = Thành thử |g(ε)| = |h(ε)| = |g(ε)| = |h(ε)| = Tóm lại, không tính tổng quát, ta giả sử |g(ε)| = Đặt t = deg(g(x)) gọi x1 , x2 , , xt nghiệm phức g(x), g(x) = (x − x1 )(x − x2 ) (x − xt ) =⇒ |g(ε)| = |(x1 − ε)(x2 − ε) (xt − ε)| Từ suy (x21 + x1 + 1)(x22 + x2 + 1) (x2t + xt + 1) = |g(ε)|2 = 18 Đặt q(x) = 2n C2kn (−1)k (x2 + x + 1)k−1 , k=1 n (x2i + xi + 1)q(xi ) + = (x2i + xi )2 + = 0, ∀i = 1, t nên ta có |g(ε)|2 q(x1 )q(x2 ) q(xt ) = (−2)t Vì q(x) ∈ Z[x] nên biểu thức đối xứng x1 , x2 , , xt số nguyên, suy biểu thức đối xứng x1 , x2 , , xt Nói riêng, q(x1 )q(x2 ) q(xt ) số nguyên Để ý C kn 2, ∀k = 1, 2n nên ta đặt n −1 q(x) = (x2 + x + 1)2 + 2.k(x) Do đó, n −1) q(x1 )q(x2 ) q(xt ) = |g(ε)|2(2 + 2.k(x1 , x2 , , xt ) = + 2.k(x1 , x2 , , xt ), với k(x1 , x2 , , xt ) biểu thức đối xứng x1 , x2 , , xt Thành thử, k(x1 , x2 , , xt ) ∈ Z q(x1 )q(x2 ) q(xt ) số lẻ Do đó, (−2)t số nguyên lẻ Đây điều vô lí, t ≥ 2.3 Chứng minh số tiêu chuẩn bất khả quy số phức Trong mục sử dụng số phức để chứng minh số kết quan trọng đa thức bất khả quy Kết tiêu chuẩn bất khả quy Eisenstein Đây kết đẹp quan trọng Nhưng tài liệu mà biết, chưa có tài liệu chứng minh số phức Sau cố gắng để tìm lời giải số phức cho toán đa thức bất khả quy kì thi chọn đội tuyển dự thi IMO năm 2013, mạnh dạn tìm chứng minh số phức cho tiêu chuẩn sau thời gian suy nghĩ, tìm chứng minh đẹp đẽ đơn giản trình bày Định lý (Tiêu chuẩn Eisenstein) Cho P (x) = an xn + an−1 xn−1 + · · · + a1 x + a0 ∈ Z[x] Nếu tồn số nguyên tố p cho điều kiện sau đồng thời thỏa mãn i) a0 , a1 , , an−1 chia hết cho p; ii) an không chia hết cho p; iii) a0 không chia hết cho p2 , 19 P (x) đa thức bất khả quy Z Chứng minh Giả sử P (x) đa thức bất khả quy Khi tồn đa thức f (x), g(x) ∈ Z[x] có bậc lớn cho P (x) = f (x)g(x), ∀x ∈ R Vì a0 = P (0) = f (0)g(0) vp (a0 ) = nên ta giả sử f (0) = b, với b | a0 , (b, p) = Đặt k = deg(f (x)) gọi x1 , x2 , , xk nghiệm (thực phức) f (x), f (x) = c(x − x1 )(x − x2 ) (x − xk ), c số nguyên c | an Ta có b = f (0) = c.(−1)k x1 x2 xk =⇒ x1 x2 xk = (−1)k b c Do x1 , x2 , , xk nghiệm P (x) nên an xni = −an−1 xin−1 + · · · + a1 xi + a0 , ∀i = 1, k Nhân k đẳng thức theo vế, ta thu k k n |an | |x1 x2 xk | = i=1 (an−1 xin−1 + · · · + a1 xi + a0 ) , hay b c akn k n = i=1 (an−1 xin−1 + · · · + a1 xi + a0 ) (1) Theo định lí Vieta, ta thấy tất đa thức x1 , x2 , , xk số hữu tỉ có mẫu chung an Vì tất đa thức đối xứng với hệ số nguyên biến x1 , x2 , , xk số hữu tỉ mà mẫu số lũy thừa an Rõ k n−1 + · · · + a1 xi + a0 ) đa thức đối xứng với hệ số nguyên ràng, i=1 (an−1 xi biến x1 , x2 , , xk nên k i=1 (an−1 xin−1 + · · · + a1 xi + a0 ) = M , |an |m (2) M, m ∈ N Hơn nữa, a0 , a1 , , an−1 p nên M p Thay (2) vào (1) n M b = akn ⇐⇒ am+k bn = M|c|n n c |an |m 20 bn p Vô lí ! Vậy điều giả ban đầu sai hay P (x) đa Từ suy am+k n thức bất khả quy Chú ý Nhờ vào công cụ số phức ta chứng minh tiêu chuẩn Eisenstein mở rộng ” Cho đa thức P (x) = an xn + an−1 xn−1 + · · ·+ a1 x+ a0 ∈ Z[x] Nếu tồn số nguyên tố p số nguyên ≤ k ≤ n cho điều kiện sau đồng thời thỏa mãn i) a0 , a1 , , ak−1 chia hết cho p; ii) ak không chia hết cho p; iii) a0 không chia hết cho p2 phân tích P (x) thành nhân tử có bậc lớn k.” Định lý (Tiêu chuẩn Cohn) Cho p số nguyên tố an an−1 a1 a0 biểu diễn p hệ số b (b ≥ 2) Khi P (x) = an xn + · · · + a1 x + a0 đa thức bất khả quy Chứng minh Để chứng minh tiêu chuẩn ta cần bổ đề sau Bổ đề Cho đa thức f (x) = an xn +· · ·+a1 x+a0 ∈ Z[x] thỏa mãn an ≥ 1, an−1 ≥ Đặt M = max |ai |, i = 1, n − Khi α nghiệm f (x), √ + + 4M Re(α) ≤ |α| < Chứng minh √ Giả sử ngược lại f (x) có nghiệm α cho Re(α) > + + 4M Vì Re(α) = Re(α) > nên |α| ≥ f (α) an−1 an−2 an−1 |an−2 | a0 |a0 | − = an + + + · · · + n ≥ an + +···+ n α α α α α |α| |α|n 1 an−1 −M +···+ ≥ Re an + α |α| |α|n 1 − |α|n−1 an−1 = an + Re(α) − M |α|2 |α|2 − |α| >1− M |α|2 − |α| − M = ≥ |α|2 − |α| |α|2 − |α| Điều vô lí, α nghiệm f (x) 21 Bổ đề Nếu đa thức f (x) = xn + an−1 xn−1 + · · · + a1 x + a0 có hệ số ∈ {0; 1} , ∀i = 1, n − 1, nghiệm α có Re(α) < Chứng minh Với n = n = 2, ta kiểm tra trực tiếp Ta xét n ≥ giả π sử ngược lại f (x) có nghiệm α với Re(α) ≥ Khi đó, arg(α) ≤ , π arg(α) > , √ √ 1+ |α| > 2.Re(α) ≥ √ > 2 √ π 1+ Vì arg(α) ≤ nên arg(α2 ) ≥ Điều vô lí, theo bổ đề |α| < Mà n ≥ nên ta có đánh giá f (α) an−1 an−2 1 ≥ 1+ + − +···+ n α α α |α| |α|n an−1 an−2 + − > 1+ α α |α| (|α| − 1) an−1 an−2 + − ≥ Re + α α |α| (|α| − 1) an−1 an−2 =1+ Re(α) + Re(α ) − > |α|2 |α|4 |α|2(|α| − 1) Vô lí, α nghiệm f (x) Ta trở lại chứng minh tiêu chuẩn Cohn Giả sử P (x) khả quy, tức tồn đa thức f (x), g(x) ∈ Z[x] có bậc lớn cho P (x) = f (x).g(x), ∀x ∈ R Vì p = P (b) = f (b).g(b) nên giả sử |f (b)| = Đặt k = deg(f (x)) gọi x1 , x2 , , xk nghiệm f (x), f (x) = c(x − x1 )(x − x2 ) (x − xk ), với c | an Xảy trường hợp sau i) Nếu b ≥ 3, với nghiệm α P (x), ta có √ √ + + 4M + 4b − |α| < ≤ , 2 22 với M = max {|ai | : i = 0, 1, , n − 2} Do đó, √ + 4b − |b − α| ≥ b − |α| > b − ≥ Vì x1 , x2 , , xk nghiệm P (x) nên = |f (b)| = |c|.|b − x1 |.|b − x2 | |b − xk | > Đây điều vô lí !Do đó, với b ≥ 3, P (x) bất khả quy ii) Nếu b = 2, |f (2)| = Theo bổ đề 2, nghiệm α P (x) thỏa mãn Re(α) < Khi ấy, |2 − α|2 − |1 − α|2 = (2 − α)(2 − α) − (1 − α)(1 − α) = − 2.Re(α) > 0, suy |2 − α| > |1 − α| Nhờ kiện này, ta có = |f (2)| = |2−x1 |.|2−x2 | |2−xk | > |1−x1 |.|1−x2 | |1−xk | = f (1) ≥ Điều mâu thuẫn kết thúc chứng minh ta Định lý (Tiêu chuẩn Perron) Cho đa thức P (x) = xn +an−1 xn−1 +· · ·+a1 x+a0 ∈ Z[x] thỏa mãn a0 = |an−1 | > + |a0 | + |a1 | + · · · + |an−2 | Khi ấy, P (x) đa thức bất khả quy Chứng minh Để chứng minh tiêu chuẩn ta cần tới bổ đề sau Bổ đề Nếu đa thức P (x) = xn + an−1 xn−1 + · · · + a1 x + a0 ∈ Z[x] thỏa mãn |an−1 | > + |a0 | + |a1 | + · · · + |an−2 |, có nghiệm thỏa mãn |z| > n − nghiệm lại thỏa mãn |z| < Chứng minh Ta có cần chứng minh cho trường hợp a0 = 0, trái lại, ta cần bỏ nhân tử có dạng xk Gọi α1 , α2 , , αn nghiệm P (x), |α1 |.|α2 | |αn | = |a0 | ≥ nên tồn nghiệm, chẳng hạn α1 cho |α1 | ≥ Mặt khác P (x) có nghiệm α với |α| = 1, |an−1 | = |an−1 αn−1| ≤ |αn + an−2 αn−2 + · · · + a1 α + a0 | ≤ |α|n + |an−2 |.|α|n−2 + · · · + |a1 |.|α| + |a0 | ≤ + |an−2 | + · · · + |a1 | + |a0 | 23 Mâu thuẫn với giả thiết Do đó, |α1 | > Đặt Q(x) = (x − α2 )(x − α3 ) (x − αn ) = xn−1 + bn−2 xn−2 + · · · + b1 x + b0 , P (x) = (x − α1 )Q(x) Bằng cách đồng hệ số, ta có an−1 = bn−2 − α1 ; a0 = −α1 b0 ; = bi−1 − α1 bi , ∀i = 1, n − Khi ấy, từ bất đẳng thức |an−1 | = |bn−2 − α1 | ≤ |bn−2 | + |α1 | |an−2 | + · · · + |a1 | + |a0 | = |bn−3 − α1 bn−2 | + · · · + |b0 − α1 b1 | + |α1 b0 | ≥ (|α1 |.|bn−2 | − |bn−3 |) + · · · + (|α1 |.|b1 | − |b0 |) + |α1 |.|b0 | = (|α1 | − 1)(|bn−2 | + |bn−3 | + · · · + |b1 | + |b0 |) + |bn−2 | với giả thiết, ta suy |bn−2 | + |bn−3 | + · · · + |b1 | + |b0 | < Bây αi (2 ≤ i ≤ n) cho |αi | ≥ 1, αi nghiệm Q(x) nên ≤ |αi |n−1 = |bn−2 αin−2 + bn−3 αin−3 + · · · + b1 αi + b0 | ≤ |bn−2 |.|αi |n−2 + |bn−3 |.|αi |n−3 + · · · + |b1 |.|αi | + |b0 | ≤ |bn−2 | + |bn−3 | + · · · + |b1 | + |b0 | < Điều vô lí kết thúc chứng minh bổ đề Trở lại với chứng minh tiêu chuẩn Perron: Giả sử P (x) khả quy, suy tồn f (x), g(x) ∈ Z[x] có bậc lớn cho P (x) = f (x)g(x), ∀x ∈ R Vì P (x) có nghiệm thoả mãn |z| > n − nghiệm lại thỏa mãn |z| < nên hai đa thức f (x) g(x) có nghiệm thỏa mãn |z| < Giả sử f (x) đa thức vậy, f (x) viết thành Khi đó, f (x) = (x − x1 )(x − x2 ) (x − xk ), k = deg(f (x)) ≤ |f (0)| = |x1 |.|x2 | |xk | < Điều vô lí chứng tỏ điều giả sử sai hay P (x) đa thức bất khả quy 24 Nhờ vào số phức chứng minh số tiêu chuẩn bất khả quy khác Hơn nữa, kết hợp số phức đa thức công cụ mạnh để giải toán đếm Tổ hợp Tuy nhiên, điều xin để dành cho chuyên đề khác Cuối số tập tự rèn luyện Bài Cho đa thức P (x) = xm + xn + với m = deg(P (x)) ≥ m.n ≡ (mod 3) Chứng minh P (x) đa thức khả quy Bài Chứng minh đa thức f (x) = (x2 − 7x + 6)2 + bất khả quy n Bài Tìm tất số nguyên tố p để đa thức f (x) = xp + x − p khả quy Bài Cho m, n ∈ Z, (n ≥ 2, m ≥ 1) p số nguyên tố Chứng minh đa thức f (x) = pm xn + an−1 xn−1 + · · · + a1 x + a0 ∈ Z[x], a0 an−1 = 0, an−1 không chia hết cho p cho pm > |a0 | + |a1 | + · · · + |an−1 |, f (x) đa thức bất khả quy Bài Cho đa thức P (x) = an xn + an−1 xn−1 + · · · + a1 x + a0 ∈ Z[x], an = Đặt : i = 1, 2, , n − Chứng minh tồn số tự nhiên M = max an m > M + cho P (m) số nguyên tố, P (x) đa thức bất khả quy Bài Cho p, q số nguyên tố phân biệt số nguyên n ≥ Tìm tất số nguyên a cho f (x) = xn + axn−1 + pq đa thức bất khả quy Bài Chứng minh với số nguyên n > 1, đa thức P (x) = xn − x − bất khả quy Tài liệu [1] Nguyễn Văn Mậu: Đa thức đại số phân thức hữu tỉ [2] Nguyễn Văn Mậu: Chuyên đề chọn lọc Số phức Áp dụng [3] Tạp chí Toán học Tuổi trẻ [4] Titu Andresscu: Problems from the book 25 [5] Victor V.Prasolov: Polynomials [6] Các nguồn tài liệu từ http://www.artofproblemsolving.com Internet: http://www.mathscope.org; [...]... một số tiêu chuẩn bất khả quy khác Hơn nữa, sự kết hợp giữa số phức và đa thức cũng là một công cụ rất mạnh để giải các bài toán đếm trong Tổ hợp Tuy nhiên, những điều này chúng tôi xin để dành cho chuyên đề khác Cuối cùng là một số bài tập tự rèn luyện Bài 1 Cho đa thức P (x) = xm + xn + 1 với m = deg(P (x)) ≥ 3 và m.n ≡ 2 (mod 3) Chứng minh rằng P (x) là đa thức khả quy Bài 2 Chứng minh rằng đa thức. .. tồn tại số tự nhiên M = max an m > M + 1 sao cho P (m) là số nguyên tố, thì P (x) là một đa thức bất khả quy Bài 6 Cho p, q là 2 số nguyên tố phân biệt và số nguyên n ≥ 3 Tìm tất cả các số nguyên a sao cho f (x) = xn + axn−1 + pq là một đa thức bất khả quy Bài 7 Chứng minh rằng với mọi số nguyên n > 1, đa thức P (x) = xn − x − 1 là bất khả quy Tài liệu [1] Nguyễn Văn Mậu: Đa thức đại số và phân thức hữu... Thành thử, k(x1 , x2 , , xt ) ∈ Z và q(x1 )q(x2 ) q(xt ) là một số lẻ Do đó, (−2)t là số nguyên lẻ Đây là điều vô lí, vì t ≥ 1 2.3 Chứng minh một số tiêu chuẩn bất khả quy bằng số phức Trong mục này chúng tôi sẽ sử dụng số phức để chứng minh một số kết quả quan trọng về đa thức bất khả quy Kết quả đầu tiên là tiêu chuẩn bất khả quy của Eisenstein Đây là kết quả rất đẹp và quan trọng Nhưng trong các... + 1 là bất khả quy n Bài 3 Tìm tất cả các số nguyên tố p để đa thức f (x) = xp + x − p khả quy Bài 4 Cho m, n ∈ Z, (n ≥ 2, m ≥ 1) và p là một số nguyên tố Chứng minh rằng nếu đa thức f (x) = pm xn + an−1 xn−1 + · · · + a1 x + a0 ∈ Z[x], trong đó a0 an−1 = 0, an−1 không chia hết cho p sao cho pm > |a0 | + |a1 | + · · · + |an−1 |, thì f (x) là một đa thức bất khả quy Bài 5 Cho đa thức P (x) = an xn +... a0 ) (1) Theo định lí Vieta, ta thấy tất cả các đa thức cơ bản của x1 , x2 , , xk đều là các số hữu tỉ có mẫu chung là an Vì thế tất cả các đa thức đối xứng với hệ số nguyên của các biến x1 , x2 , , xk cũng là số hữu tỉ mà mẫu số là lũy thừa nào đó của an Rõ k n−1 + · · · + a1 xi + a0 ) là một đa thức đối xứng với hệ số nguyên ràng, i=1 (an−1 xi của các biến x1 , x2 , , xk nên k i=1 (an−1... một nghiệm thoả mãn |z| > 1 và n − 1 nghiệm còn lại thỏa mãn |z| < 1 nên một trong hai đa thức f (x) hoặc g(x) chỉ có nghiệm thỏa mãn |z| < 1 Giả sử f (x) là đa thức như vậy, thì f (x) có thể viết thành Khi đó, f (x) = (x − x1 )(x − x2 ) (x − xk ), k = deg(f (x)) 1 ≤ |f (0)| = |x1 |.|x2 | |xk | < 1 Điều vô lí này chứng tỏ điều giả sử là sai hay P (x) là một đa thức bất khả quy 24 Nhờ vào số phức... năm 2014: ”Cho đa thức P (x) = (x2 − 7x + 6)2n + 13, n ∈ N∗ Chứng minh rằng P (x) không thể phân tích thành tích của n+ 1 đa thức khác hằng với hệ số nguyên.” 12 ii) Nếu thay 13 bởi một số nguyên tố p sao cho p ≡ ±1 (mod 5), thì kết quả vẫn không thay đổi Ví dụ 6 Cho n là các số nguyên dương và p là một số nguyên tố Chứng minh rằng đa thức f (x) = xp − x + pn bất khả quy Giải Giả sử f (x) khả quy,... tôi biết, thì chưa có một tài liệu nào chứng minh bằng số phức Sau khi cố gắng để tìm một lời giải bằng số phức cho bài toán về đa thức bất khả quy trong kì thi chọn đội tuyển dự thi IMO năm 2013, chúng tôi đã mạnh dạn tìm một chứng minh bằng số phức cho tiêu chuẩn này và sau một thời gian suy nghĩ, chúng tôi đã tìm được một chứng minh đẹp đẽ và khá là đơn giản như trình bày dưới đây Định lý 1 (Tiêu... nghiệm của g(x) nên (x2i − 7xi + 6)2n = −13 Thành thử, (x21 − 7x1 + 6)(x22 − 7x2 − 6) (x2k − 7xk + 6) 2n = 13k Như thế, g 2n (1).g 2n (6) = 13k Vì 13 là số nguyên tố nên k = 2n, suy ra |g(1)| |g(6)| = 13 Nhưng 5 | g(6) − g(1) nên g(6) = g(1) Suy ra 13 = g 2 (1) Vô lí ! Chú ý Ta có vài chú ý sau về bài toán này i) Bài toán trên là một kết quả mạnh hơn bài toán trong kì thi VMO năm 2014: ”Cho đa. .. thức khả quy Vậy điều giả sử là sai hay f (x) phải là một đa thức bất khả quy Ví dụ 7 Cho p là số nguyên tố và a là số nguyên không chia hết cho p Chứng minh rằng P (x) = xp − x + a bất khả quy Giải Giả sử P (x) khả quy Khi ấy tồn tại Q(x), H(x) ∈ Z[x] có bậc lớn hơn hoặc bằng 1 sao cho P (x) = Q(x).H(x) Đặt k = deg(Q(x)) và gọi x1 , x2 , , xk là tất cả các nghiệm phức của Q(x) Ta có xpi = xi − ... Nghiên cứu tính chất sâu đa thức ii) Đề xuất số biện pháp rèn luyện tư duy, lực giải toán kĩ vận dụng số phức để giải toán đa thức như: chia đa thức, toán xác định đa thức, toán đa thức bất khả... đối xứng với hệ số nguyên biến x1 , x2 , , xn nhận giá trị nguyên (tương ứng: hữu tỉ) 2.1 Sử dụng số phức giải toán đa thức Bài toán xác định đa thức, chia hết Ví dụ Tìm tất đa thức P (x) ∈ R[x]... với hệ số thuộc K Định lý Mọi đa thức bậc n (n ≥ 0) với hệ số thực có tối đa n nghiệm thực (tính bội) Định lý Mọi đa thức bậc n (n ≥ 1) có n nghiệm phức (tính bội) Định lý (Định lí Bezout) Số thực

Ngày đăng: 15/12/2015, 00:20

Từ khóa liên quan

Mục lục

  • Lí do chon tài

  • Muc ích và nhim vu nghiên cu

  • Ði tng hoc sinh

  • Kin thc chun bi

  • S dung s phc giai các bài toán v a thc

    • Bài toán xác inh a thc, chia ht

    • Chng minh a thc bt kha quy

    • Chng minh mt s tiêu chun bt kha quy bng s phc

Tài liệu cùng người dùng

Tài liệu liên quan